Difference between revisions of "2006 iTest Problems/Problem 31"

(Solution to Problem 31 -- Infinite Summation of a Rational Function)
 
m (Solution)
 
Line 10: Line 10:
 
<cmath>\sum_{n=2}^\infty\dfrac{1}{n^2 - 1} + \dfrac{n}{n^4 + 2n^2 + 1 - n^2}</cmath>
 
<cmath>\sum_{n=2}^\infty\dfrac{1}{n^2 - 1} + \dfrac{n}{n^4 + 2n^2 + 1 - n^2}</cmath>
 
<cmath>\sum_{n=2}^\infty\dfrac{1}{(n+1)(n-1)} + \dfrac{n}{(n^2 + n + 1)(n^2 - n + 1)}</cmath>
 
<cmath>\sum_{n=2}^\infty\dfrac{1}{(n+1)(n-1)} + \dfrac{n}{(n^2 + n + 1)(n^2 - n + 1)}</cmath>
In the fraction <math>\frac{1}{(n+1)(n-1)}</math>, we can split the fraction as <math>\frac{1}{2(n-1)} - \frac{1}{2(n+1)}</math>, making <math>\sum_{n=2}^\infty\dfrac{1}{(n+1)(n-1)}</math> a [[telescoping]] series. Plugging in values results in <math>\frac12 - \frac16 + \frac14 - \frac18 + \frac16 - \frac10 + \cdots = \frac12 + \frac14 = \frac34</math>.
+
In the fraction <math>\frac{1}{(n+1)(n-1)}</math>, we can split the fraction as <math>\frac{1}{2(n-1)} - \frac{1}{2(n+1)}</math>, making <math>\sum_{n=2}^\infty\dfrac{1}{(n+1)(n-1)}</math> a [[telescoping]] series. Plugging in values results in <math>\frac12 - \frac16 + \frac14 - \frac18 + \frac16 - \frac1{10} + \cdots = \frac12 + \frac14 = \frac34</math>.
  
 
<br>
 
<br>

Latest revision as of 20:47, 11 December 2018

Problem

The value of the infinite series \[\sum_{n=2}^\infty\dfrac{n^4+n^3+n^2-n+1}{n^6-1}\] can be expressed as $\tfrac pq$ where $p$ and $q$ are relatively prime positive numbers. Compute $p+q$.

Solution

Notice that $n^6 - 1 = (n^2 - 1)(n^4 + n^2 + 1)$, and notice that the numerator contains $n^4 + n^2 + 1$. Also, notice that $n^3 - n = n(n^2 - 1)$. We can rewrite the expression by factoring and splitting the fractions. \[\sum_{n=2}^\infty\dfrac{n^4+n^2 + 1 + n(n^2 - 1)}{(n^2 - 1)(n^4 + n^2 + 1)}\] \[\sum_{n=2}^\infty\dfrac{1}{n^2 - 1} + \dfrac{n}{n^4 + n^2 + 1}\] \[\sum_{n=2}^\infty\dfrac{1}{n^2 - 1} + \dfrac{n}{n^4 + 2n^2 + 1 - n^2}\] \[\sum_{n=2}^\infty\dfrac{1}{(n+1)(n-1)} + \dfrac{n}{(n^2 + n + 1)(n^2 - n + 1)}\] In the fraction $\frac{1}{(n+1)(n-1)}$, we can split the fraction as $\frac{1}{2(n-1)} - \frac{1}{2(n+1)}$, making $\sum_{n=2}^\infty\dfrac{1}{(n+1)(n-1)}$ a telescoping series. Plugging in values results in $\frac12 - \frac16 + \frac14 - \frac18 + \frac16 - \frac1{10} + \cdots = \frac12 + \frac14 = \frac34$.


In the fraction $\frac{n}{(n^2 + n + 1)(n^2 - n + 1)}$, plugging in the first few values results in $\frac{2}{7 \cdot 3} + \frac{3}{13 \cdot 7} + \frac{4}{21 \cdot 13} + \cdots$. The structure suggests that we could try to model the same fraction with a telescoping series.


Substituting $n$ for $a+1$ in $n^2 - n + 1$ results in $a^2 + a + 1$. Also, because $\tfrac12 \cdot (n^2 + n + 1) - \tfrac12 \cdot (n^2 - n + 1) = n$, we find that the expression is equal to $\frac{1}{2(n^2 - n + 1)} - \frac{1}{2(n^2 + n + 1)}$, confirming that $\sum_{n=2}^\infty\dfrac{n}{(n^2 + n + 1)(n^2 - n + 1)}$ is a telescoping series. The expression equals $\frac{1}{6} - \frac{1}{14} + \frac{1}{14} - \frac{1}{26} + \cdots = \frac{1}{6}$.


Therefore, $\sum_{n=2}^\infty\dfrac{n^4+n^3+n^2-n+1}{n^6-1} = \frac34 + \frac16 = \frac{11}{12}$, so $m+n = \boxed{23}$.

See Also

2006 iTest (Problems, Answer Key)
Preceded by:
Problem 30
Followed by:
Problem 32
1 2 3 4 5 6 7 8 9 10 11 12 13 14 15 16 17 18 19 20 21 22 23 24 25 26 27 28 29 30 31 32 33 34 35 36 37 38 39 40 U1 U2 U3 U4 U5 U6 U7 U8 U9 U10